2014 dxdy logo

Научный форум dxdy

Математика, Физика, Computer Science, Machine Learning, LaTeX, Механика и Техника, Химия,
Биология и Медицина, Экономика и Финансовая Математика, Гуманитарные науки




Начать новую тему Ответить на тему На страницу Пред.  1, 2, 3  След.
 
 Re: Свежее неравенство
Сообщение06.03.2010, 21:45 
Заслуженный участник


26/06/07
1929
Tel-aviv
ShMaxG в сообщении #295267 писал(а):
Теперь пользуемся $\[{a^2}{b^2} + {a^2}{c^2} + {b^2}{c^2} + 2{a^4}{b^2}{c^2} + 2{b^4}{a^2}{c^2} + 2{c^4}{a^2}{b^2} = 9{v^4}\]$

Здесь Вы немного переборщили... :wink:
и, может быть, как следствие здесь есть небольшая ошибка:
ShMaxG в сообщении #295267 писал(а):
Далее,...
$
  {t^3} + \left( {2 - 12u} \right)t + 9u{v^4} \geqslant 0 $

После исправления можно что-то увидеть...

 Профиль  
                  
 
 Re: Свежее неравенство
Сообщение06.03.2010, 22:07 
Заслуженный участник
Аватара пользователя


11/04/08
2740
Физтех
Теперь воспользуемся $\[\left( {{a^2}{b^2} + {a^2}{c^2} + {b^2}{c^2}} \right) + 2{a^2}bc + 2{b^2}ac + 2{c^2}ab = 9{v^4}\]$:

Получим $\[\frac{2}
{{{w^3}}} - \frac{{12{u^2}{w^3} - 9u{v^4}}}
{{{w^6}}} \geqslant  - {w^3}\]$

А теперь замена $t=w^3$
$
\[{t^3} + 2\left( {1 - 6{u^2}} \right)t + 9u{v^4} \geqslant 0\]$

 Профиль  
                  
 
 Re: Свежее неравенство
Сообщение06.03.2010, 22:07 


21/06/06
1721
Аркадий, а хотелось бы узнать, а есть ли у Вас решение этого неравенства, основанное исключительно на классике, ну там AM-GM, Коши, Чебышев, Гельдер и т.п.?

 Профиль  
                  
 
 Re: Свежее неравенство
Сообщение06.03.2010, 22:11 
Заслуженный участник


26/06/07
1929
Tel-aviv
Sasha2 в сообщении #295292 писал(а):
Аркадий, а хотелось бы узнать, а есть ли у Вас решение этого неравенства, основанное исключительно на классике, ну там AM-GM, Коши, Чебышев, Гельдер и т.п.?

Честно говоря, много об этом не думал. Может, и можно.
ShMaxG, теперь всё верно!

 Профиль  
                  
 
 Re: Свежее неравенство
Сообщение06.03.2010, 22:51 
Заслуженный участник
Аватара пользователя


11/04/08
2740
Физтех
Фишка в том, что при некоторых $u,t,v$ это неравенство может нарушаться, но им не будут соответствовать $a,b,c$, определенные на условиях задачи... В общем виде даже не знаю, как это можно доказать...

 Профиль  
                  
 
 Re: Свежее неравенство
Сообщение07.03.2010, 00:08 
Заслуженный участник
Аватара пользователя


11/04/08
2740
Физтех
При фиксированном $u$, функция $\[f\left( t \right) = {t^3} - 2\left( {6{u^2} - 1} \right)t + 9u{\left( {\frac{{3{u^2} - 1}}
{2}} \right)^2}\]$ убывает при $t \in [0,1]$. И ее минимальное значение реализуется при $t=1$.
Тогда (в силу "независимости" $u$ и $t$) остается рассмотреть получившуюся функцию $u$:
$\[g\left( u \right) = 1 - 2\left( {6{u^2} - 1} \right) + 9u{\left( {\frac{{3{u^2} - 1}}
{2}} \right)^2}\]$, которая, в свою очередь, неположительна на всем $u \in [\frac{1}{\sqrt{3}},1]$, и равна нулю только при $u=1$.

 Профиль  
                  
 
 
Сообщение07.03.2010, 00:45 
Заслуженный участник


26/06/07
1929
Tel-aviv
ShMaxG в сообщении #295325 писал(а):
При фиксированном $u$, функция $\[f\left( t \right) = {t^3} - 2\left( {6{u^2} - 1} \right)t + 9u{\left( {\frac{{3{u^2} - 1}}
{2}} \right)^2}\]$ убывает при $t \in [0,1]$.

Вот оно!
ShMaxG в сообщении #295325 писал(а):
И ее минимальное значение реализуется при $t=1$.

А вот это неверно!

 Профиль  
                  
 
 Re: Свежее неравенство
Сообщение07.03.2010, 01:08 
Заслуженный участник
Аватара пользователя


11/04/08
2740
Физтех
А, ну да, минимум в точке $\[{t^*} = \sqrt {\frac{2}
{3}\left( {6{u^2} - 1} \right)}  \in \left[ {\sqrt {\frac{2}
{3}} ,\sqrt {\frac{{10}}
{3}} } \right]\]
$.
Пусть для начала $\[{t^*} \geqslant 1\]$, тогда минимум достигается в $t=1$.

Учтем:

$\[\left\{ \begin{gathered}
  abc = 1 \hfill \\
  a + b + c = 3u \hfill \\
  ab + bc + ac = 3\frac{{3{u^2} - 1}}
{2} \hfill \\ 
\end{gathered}  \right.\]$

Это все даст уравнение на $a$:

$\[{a^3} - 3{a^2}u + \frac{{9{u^2} - 3}}
{2}a - 1 = 0\]$

Так же у нас есть очень важное условие: $\[{\left( {3u - a} \right)^2} \geqslant \frac{4}
{a}\]
$.

Таким образом имеем:

$\[\left\{ \begin{gathered}
  {a^3} + 9{u^2}a - 6u{a^2} - 4 \geqslant 0 \hfill \\
  2{a^3} - 6{a^2}u + 9{u^2}a - 3a - 2 = 0 \hfill \\ 
\end{gathered}  \right.\]$

Что приводит нас к $\[{a^3} - 3a + 2 \leqslant 0 \Leftrightarrow a = 1\]$. Что мгновенно дает $u=1$. А это дает $a=b=c=1$. В этом случае неравенство выполнено.

Остается рассмотреть случай оставшихся значений $t^*$.

 Профиль  
                  
 
 Re: Свежее неравенство
Сообщение07.03.2010, 01:08 


21/06/06
1721
Аркадий (извиняюсь заранее, что в эту тему влезаю с данным воросом), а подскажите пожалуйста вот такой момент:
В униге Пам Ким Хана "Секреты неравенств" приводится такое неравенство
Доказать, что $a^2b^2+b^2c^2+a^2c^2 \le 2$, если $a, b, c$ неотрицательные числа с суммой равной 2.

Тогда имеем $(a+b+c)^2=a^2+b^2+c^2+2ab+2bc+2ac=4$
А значит $3(ab+bc+ac) \le 4$
Возводя это последнее неравенство в квадрат легко получаем, что $a^2b^2+b^2c^2+a^2c^2 \le \frac{16}{9}$, что лучше чем 2. Но что-то слишком просто. Не могу понять, ошибка у меня?

 Профиль  
                  
 
 Re: Свежее неравенство
Сообщение07.03.2010, 01:42 
Заслуженный участник


26/06/07
1929
Tel-aviv
Sasha2 в сообщении #295338 писал(а):
Аркадий (извиняюсь заранее, что в эту тему влезаю с данным воросом), а подскажите пожалуйста вот такой момент:
В униге Пам Ким Хана "Секреты неравенств" приводится такое неравенство
Доказать, что $a^2b^2+b^2c^2+a^2c^2 \le 2$, если $a, b, c$ неотрицательные числа с суммой равной 2.


По-моему, верно даже $a^2b^2+b^2c^2+a^2c^2 \le 1$ при тех же ограничениях. :wink:

 Профиль  
                  
 
 Re: Свежее неравенство
Сообщение07.03.2010, 02:18 


21/06/06
1721
Да точно, у него там опечаток куча. Разобрался спасибо.

 Профиль  
                  
 
 Re: Свежее неравенство
Сообщение07.03.2010, 13:32 
Заслуженный участник
Аватара пользователя


11/04/08
2740
Физтех
Ситуация такая.

Берем любой возможный набор $(a,b,c)$. Им соответствует $u$. Вычисляем $t^*$. И проверяем неравенство на этой $u,t^*,v$, причем $v$ находим из условия принадлежности сфере. Если неравенство выполнено, то можем утверждать, что оно тем более будет выполнено при других $(a,b,c)$, которым отвечает та же самая $u$, но остальные $t$.

Выше я рассмотрел фактически случай всех $u \in \left[\sqrt{\frac{2}{15}},1 \right]$. Либо это удивительно, либо это я ошибаюсь, но получается что я доказал справедливость неравенства для всех таких $u$, разобрав только один случай, $u=1$.

 Профиль  
                  
 
 Re: Свежее неравенство
Сообщение07.03.2010, 19:20 


21/06/06
1721
Вот еще в связи с этим неравенством:
Его можно переписать так:
$\frac{a}{b^2}+\frac{a}{c^2}+\frac{b}{a^2}+\frac{b}{c^2}+\frac{c}{a^2}+\frac{c}{b^2}+3abc \ge (a+b+c)+(\frac{a^2}{b}+\frac{a^2}{c}+\frac{b^2}{a}+\frac{b^2}{c}+\frac{c^2}{a}+\frac{c^2}{b})$
Это после замены 3 в правой части на $(a^2+b^2+c^2)$

Если обозначить:
1) $LHS=\frac{a}{b^2}+\frac{a}{c^2}+\frac{b}{a^2}+\frac{b}{c^2}+\frac{c}{a^2}+\frac{c}{b^2}+3abc$
2) $M=(a+b+c)$
3) $N=\frac{a^2}{b}+\frac{a^2}{c}+\frac{b^2}{a}+\frac{b^2}{c}+\frac{c^2}{a}+\frac{c^2}{b}$


То вот никак не могу допереть, можно ли подобрать так p и q, что $p\times LHS+q\times M$ по AM-GM стало больше некоторого $r \times N$
И аналогично $s\times LHS+t\times N$ стало бы больше $r \times M$. Или так в пригципе невозможно уравновесить эти обе части.

P.S. Идея навеяна разбором методом доказательства одного неравенства из книги Пама "Секреты неравенств".

 Профиль  
                  
 
 Re: Свежее неравенство
Сообщение08.03.2010, 08:14 
Заслуженный участник


26/06/07
1929
Tel-aviv
Вот моё доказательство.
Мы уже выяснили, что наше неравенство эквивалентно $f(w^3)\geq0$, где $f(w^3)= w^9 - 2(3u^2 +2v^2)w^3 + 9uv^4 $ и $f$ убывает.
Поэтому $f$ при фиксированных $u$ и $v^2$ достигает своего наименьшего значения при наибольшем значении $w^3$.
Но $a,$ $b$ и $c$ - действительные корни уравнения $w^3=x^3-3ux^2+3v^2x$.
Поэтому наибольшее значение $w^3$ получит, когда два из корней этого уравнения совпадут.
Таким образом, осталось проверить наше неравенство для $b=c=1$ в гомогенизированном варианте.
Но в этом случае $f(w^3)\geq0\Leftrightarrow w^9-2(3u^2+2v^2)w^3+9uv^4\geq0\Leftrightarrow$
$\Leftrightarrow w^9 - 2(3u^2 + 2v^2)(3u^2 - 2v^2)^2w^3 + 9uv^4(3u^2 - 2v^2)^2\geq0\Leftrightarrow$
$\Leftrightarrow a^3 - 2\left(\frac {1}{3}(a + 2)^2 + \frac {2}{3}(2a + 1)\right)\left(\frac {1}{3}(a + 2)^2 - \frac {2}{3}(2a + 1)\right)^2a +$
$+ \frac {1}{3}(a + 2)(2a + 1)^2\left(\frac {1}{3}(a + 2)^2 - \frac {2}{3}(2a + 1)\right)^2\geq0\Leftrightarrow$
$\Leftrightarrow(a-1)^2(2a^5+3a^3-8a^2+4a+8)\geq0$, что очевидно.

 Профиль  
                  
 
 Re: Свежее неравенство
Сообщение15.03.2010, 22:03 
Заслуженный участник
Аватара пользователя


11/01/06
3822
Можно и с помощью brute force доказать.
Надо доказать неравенство
$$\sum_{cyc}\frac{b+c}{a^2}-3\left(\frac1a+\frac1b+\frac1c\right)+\frac{27abc}{(a^2+b^2+c^2)^2}\ge0.$$
Перепишем его в виде
$$\sum_{cyc}\frac{(a-b)^2(2a^5-3a^3b^2+4a^3c^2+a^2b^3-9a^2b^2c+4a^2bc^2+2ac^4+2b^5+4b^3c^2+2bc^4)}{2a^2b^2(a^2+b^2+c^2)^2}\ge0.$$
Таким образом, достаточно доказать, что
$$2a^5-3a^3b^2+4a^3c^2+a^2b^3-9a^2b^2c+4a^2bc^2+2ac^4+2b^5+4b^3c^2+2bc^4\ge0.$$
Это неравенство выполняется с большим запасом. Например, достаточно сложить неравенства
$$\frac95a^5+\frac65b^5\ge3a^3b^2,$$
$$a^2b^3+4a^2bc^2\ge4a^2b^2c,$$
$$\frac15a^5+4a^3c^2+\frac45b^5+4b^3c^2\ge4\left(\frac15a^5\cdot4a^3c^2\cdot\frac45b^5\cdot4b^3c^2\right)^{1/4}>5a^2b^2c.$$

 Профиль  
                  
Показать сообщения за:  Поле сортировки  
Начать новую тему Ответить на тему  [ Сообщений: 33 ]  На страницу Пред.  1, 2, 3  След.

Модераторы: Модераторы Математики, Супермодераторы



Кто сейчас на конференции

Сейчас этот форум просматривают: нет зарегистрированных пользователей


Вы не можете начинать темы
Вы не можете отвечать на сообщения
Вы не можете редактировать свои сообщения
Вы не можете удалять свои сообщения
Вы не можете добавлять вложения

Найти:
Powered by phpBB © 2000, 2002, 2005, 2007 phpBB Group